return to table of content

Webb and Hubble confirm Universe's expansion rate

arcastroe
68 replies
20h0m

I've asked this question before, but I don't think I received a good answer, so figured I'd try asking again.

How do we know that galaxies are accelerating away from us and not moving at constant speed? People often point to the observation that the further away a galaxy is, the faster it appears to be moving away from us, implying acceleration.

However, wouldn't we expect to see the same observation even without any acceleration? Imagine there are some objects in space all moving in random directions and speeds, relative to Earth. After long enough time, all objects will appear to be moving away from Earth, even if they were moving towards it initially. And after long enough time, the objects that move fastest should be farthest away, by the simple definition of speed!

In short, even if galaxies weren't accelerating, we would still see that the further away a galaxy is, the faster it recedes.

narag
14 replies
17h45m

How do we know that galaxies are accelerating away from us and not moving at constant speed?

There's a more basic question: How do we know the galaxies are moving? It seems (I haven't seen any other response, like... ever) that we have one and only one way to measure the speed of galaxies: the red shift.

It's impossible to triangulate those huge distances and the time scale would also be a barrier, so no way of confirming the red shift calculations with a different method. That means that if the red shift was caused by any other effect, say the light "degrading" after millions of years of travelling the void, all the calculations would be invalid.

I've asked about this many times and the answers are in the line of "we don't know any other reason for the light to red shift" and "the current theoretical frame is consistent", even if there isn't any other measure to be consistent with.

There was a prediction (expansion is related to Big Bang) that the far away galaxies, being younger, would have a different composition. This prediction seems to be failing, but advances in instrumental could give us a more precise answer in the future.

k7sune
4 replies
16h35m

“Degrading” sounds very intuitive to me. Can the frequency of the light waves simply slow down over a very long distance/time? Or maybe the speed of light simply slows down over an unimaginably long distance? We don’t have any model to describe such behavior, but everyday objects around us all slow down one way or another, what makes light so different?

Sprocklem
3 replies
13h42m

IIRC, this was one of the explanations proposed when the existence of a red shift was first noted: that the light is somehow slowly losing its energy over very long distances, becoming “redder” as it did so. It ultimately lost out to the dark energy / space-time expansion theory, although I do not recall why. Presumably there was some observation that precluded “degrading” light from being the sole explanation.

semi-extrinsic
1 replies
11h42m

There are several challenges for "tired light", or indeed any theory that's an alternative to expansion of the universe.

The theory has to explain why the light gets redshifted, but does not get blurred, and the spectral lines do not get broadened. This severely restricts the type of interactions possible. Also the theory has to explain the consistency between redshifts within our own galaxy, to that of far-away galaxies.

pigpang
0 replies
9h33m

My theory is that redshifting caused by gravitational background noise. I did calculation somewhere on HN or Youtube few months ago and numbers are of same magnitude.

javajosh
1 replies
14h55m

>It's impossible to triangulate those huge distances

Galaxies are BIG. Andromeda is faint, but the same angular size as the moon. It's 2.5Mly away, but it's also 150kly across. Over a long enough time line you could do triangulation on it. In fact it's moving toward our galaxy, but very, very slowly compared with its diameter at 110kps. But yeah, in theory you could do triangulation on it over a very long period of time.

mr_toad
0 replies
5h27m

For triangulation to work you need to move, not your target. Triangulation is only used for objects within about 1000 parsecs, where we can triangulate using the movement of the Earth along its orbit.

garrettgarcia
1 replies
14h16m

The American astronomer, Halton Arp, had a theory he called "intrinsic redshift". My limited understanding is that he saw evidence of "very close" and "very far away" structures that are connected to each other in space, which makes no sense. He theorized that redshift may also be indicative of the age of a galaxy, rather than only indicating velocity.

The interpretation of redshift as velocity is also the primary reason cosmologists think the universe is expanding.

the-mitr
0 replies
11h48m

His book Seeing Red which raises several interesting and troublesome questions for the standard big bang model is worth a read.

__turbobrew__
1 replies
14h14m

Degrading light sounds like an interesting idea to me, you could call it the “cosmological damper” if you will. Thought experiment: imagine you have light particles in a perfectly circular orbit around a black hole. Does the light ever fall into the black hole or does it orbit for eternity?

oneshtein
0 replies
12h24m

There a whole set of theories, called "Tired light" theories, which tries to explain Cosmological Red Shift by degradation of photons with time. Buy they require whole set of different cosmological principles: a medium for light propagation is required to dump lost energy into, no Big Bang. But even with a tired light theory, galaxies are accelerating toward attractors, see https://arxiv.org/pdf/1702.02483v1.pdf https://www.youtube.com/watch?v=NpV0GQo3P0c

ganzuul
0 replies
10h42m

"we don't know any other reason for the light to red shift"

https://en.wikipedia.org/wiki/G%C3%B6del_metric

There are versions of this which do provide another reason for red shift.

Personally I keep this in mind as a means to free my thinking from a single narrative.

at_a_remove
0 replies
14h0m

You're not going to get a simple answer because the answer is quite complex.

Astronomy is the paleontology of photons. You should take an astronomy course if you really want to know, but essentially a "ladder" was built of distances, starting with the very near and slowly building outward using various techniques and discoveries of physics as they became available. This is called the cosmic distance ladder. You start with stellar parallax, then after that you go farther with "standard candles" (particular types of variable stars). But then you have to get even further out, where you can no longer see an individual star, and then you rely on specific breeds of supernovae. Only then do you get to redshift, and compounding tons of data from step three seems to verify the redshift estimates. By the time you get to the Hubble constant, it was a huge rift between two communities over what was still a factor of two difference.

It's quite fascinating, but I can't really dump out an entire book into a comment.

runeb
13 replies
19h29m

Their light is more red shifted the farther away they are. I'm no expert on this, but I believe in a constant-speed scenario they would have equal red shift no matter the distance

willis936
11 replies
19h24m

The assumption made here is that relative velocity is the only method that would redshift light. Gravitational redshift is a thing and our model of gravity is incomplete.

pdonis
10 replies
17h22m

> The assumption made here is that relative velocity is the only method that would redshift light.

Not in our actual model of the universe, no. The redshift of light is determined by the spacetime geometry and the worldlines of the emitter and receiver. That is a general formula that works in any spacetime.

> Gravitational redshift is a thing

Not for the universe as a whole, no. Gravitational redshift is only meaningful in certain kinds of spacetimes, namely stationary spacetimes (which, roughly speaking, describe objects that either don't change with time at all, or which are periodic, like a rotating planet or star). The spacetime that describes our universe as a whole is not stationary and there is no meaningful concept of gravitational redshift.

> our model of gravity is incomplete

In the sense that we do not have a quantum theory of gravity, yes. But that does not affect anything under discussion here. Our current theory of gravity, GR, works fine for treating the expansion of the universe and whether or not it is accelerating.

willis936
9 replies
17h13m

Our current theory of gravity, GR, works fine for treating the expansion of the universe and whether or not it is accelerating.

Then why are there phantoms in the data that need dark matter and dark energy to make the supposed working model fit them?

pdonis
8 replies
17h5m

I'm not sure what you mean by "phantoms in the data". The distribution of stress-energy is a free parameter in GR; it has to be inferred from observations.

The terms "dark matter" and "dark energy" are just names for, respectively, "stress-energy that acts like the matter we can see, but we can't see it", and "stress-energy that acts like a cosmological constant". Neither of those things poses any problem for GR, since both types of stress-energy are allowed for in the theory.

"Dark matter" poses a problem for particle physicists, who have so far been unable to find any fundamental particles that would produce the observed properties. "Dark energy" only poses a problem if for some reason you don't like having a nonzero cosmological constant.

willis936
7 replies
16h49m

"Dark matter" poses a problem for particle physicists, who have so far been unable to find any fundamental particles that would produce the observed properties.

It's clear to me why we haven't made any new discoveries in cosmology in the past two decades. It's this exact attitude of "the model is the truth". All models are wrong. The data can help you improve it, but you have to at least want to improve it.

sophacles
2 replies
14h55m

What do you think they do now?

How do you propose they do it differently?

What evidence do you have that what they are doing now doesn't work, and does the all the evidence of how they work support your hypothesis?

Be detailed, because your comment just has some motivational speaker nonsense but no depth. For example, in the last 20 years cosmology has:

+ Refined its model of stellar formation based on observational data of the number of planets found observationally, and used this to validate and invalidate several model adjustments.

+ Observed galaxies that appear not to have dark matter, and by their existence and behavior validate some theories of dark matter, and validated others, which predict such galactic behavior. (e.g. some theories attempting to update gravity).

+ Run simulations of stellar and glactic formation that predicted structures in the universe that were later observed.

Everywhere they look they are finding things the models don't explain well, and refining the models - that is literally using the data to improve the models.

If you think you can come up with something better, then do it - all you gotta do is make up some mumbo jumbo and write down any old equation. It probably should:

- provide the same results as were observed when the plugging in the experimental parameters of existing experiments.

- explain "wierd stuff" in the data that existing models couldn't.

- predict future observations of the known phenomena with the same or better accuracy as the old model

- predict currently unobserved and unpredicted phenomena

Go ahead and take a stab real quick - I'm sure you can do it. I mean Gallieo did it, so did Newton and Einstein. Next up is willis936.

hnfong
1 replies
12h46m

People can be right about problems with a process or way of thinking without being the next Einstein. There's no need to get personal.

sophacles
0 replies
3h53m

But in this case you and your friend are not right about the process. There was a claim that "the models aren't being updated based on the new data" which is categorically false. It's not that they pointed out problems in the process, it's that they flat out lied about things - such dishonesty doesn't help solve any problems you imagine you see, its just trolling.

jMyles
2 replies
12h51m

we haven't made any new discoveries in cosmology in the past two decades

Is that true?

At least for my hobbyist understanding of the progress of cosmology, quite a lot seems to have happened in the past two decades. Confirmation of the Higgs Boson at CERN [0] kept me up all night to watch the press conference; I found it extremely exciting. (Maybe you count this strictly as observational particle physics and not cosmology, but I might appeal for it to be allowed in the context of your critique).

And what of TFA? Isn't what we're reading now a new discovery in cosmology?

What about the rush of exoplanet discoveries?

What about the dramatically different galactic properties now observed in increasingly strange corners of the observable universe (including some which perhaps give insight into some of the properties of "dark matter" or whatever it ends up being)?

0: https://home.web.cern.ch/news/news/physics/new-results-indic...

oneshtein
1 replies
12h17m

Yes, it's true. Mainstream science refuses to accept anything radically new because of huge baggage. Nobody wants to look stupid, then relearn, recalculate, republish, reteach everything, or lose their tenures, grants, etc. It's why science advances in small incremental steps. AFAIK, there is a team of scientists secretly working on radically new set of theories (I got contact but cannot join because of war).

mr_mitm
0 replies
8h8m

That notion is ridiculous. Finding something radically new is every scientist's dream. Look at how Einstein is perceived, who arguably found one of the most radically new theories. Nobody thought he looked stupid or lost tenure. No one goes into science hoping to simply confirm what everybody already thought was true.

mr_mitm
0 replies
8h1m

The true reason we have made little progress in the past 20 or so years (and a 20 year slouch is historically nothing unusual) is that pretty much all data we collected in that time frame confirmed the standard model. It's the one big dilemma cosmology has. The standard model (LambdaCDM) works unreasonably well. Our problems with it are largely theoretical. New data is also hard to come by. Look at how long it took to plan, build and launch Euclid, cosmology's big hope of finding new physics. The hubble tension from the OP's article is already the most interesting discovery since 1998 when evidence for dark energy was first seen.

And trust me, all scientists know that all models are wrong. This isn't some unique insight that is beholden to amateur scientists on the sidelines.

px43
0 replies
19h8m

As I understand it, if the expansion was constant, farther away stuff would still be more red shifted. Stuff twice as far away appears to be moving twice as fast. It helps me to imagine the expansion of a metal cookie sheet, where the two edges are moving apart faster relative to each other compared to the speed that they're moving away from the center.

The surprising bit is that the far away stuff seems to be even more red shifted than that, so we're not just expanding, but the rate of expansion seems to be accelerating.

digging
11 replies
19h16m

It's because the movement is ascribed to the expansion of space itself, not the individual galaxies. We don't have any reason to believe galaxies are moving in random directions at random speeds (not at scales that explain the redshifts we call the expansion of the universe).

In your explanation, I think we'd expect to see some very distant, very slow-moving galaxies moving toward us. And there may be some very fast-moving galaxies close to us that just started really far away. Objects would be entering our local universe from outside it, and that simply doesn't happen.

arcastroe
10 replies
18h36m

In your explanation, I think we'd expect to see some very distant, very slow-moving galaxies moving toward us

Thank you, but I suppose I'm not really questioning the big bang piece. My question was mostly in regards to the continued acceleration piece. Feel free to disregard the "in random directions" part of my original post.

I'm picturing more of an explosion in empty space. A firework or granade of sorts. Any individual dust/shard of the explosion still sees all other objects moving away from it and the rest of my question stands. But I suppose this would imply a "center" to the explosion, which I've also heard is not the case.

Theres a few other comments offering more clarity to the acceleration piece. Thank you everyone!

pdonis
9 replies
17h26m

> I'm picturing more of an explosion in empty space.

No, that's not what the big bang is.

> this would imply a "center" to the explosion, which I've also heard is not the case

That's correct. The big bang does not work like anything ordinary that you are used to imagining. The math is straightforward and unambiguous, but there is no good ordinary language description that corresponds to the math.

lp4vn
7 replies
15h20m

As I understand it, before the big bang the whole observable universe was contained in a small sphere and then it started to expand metrically. Is this interpretation correct?

Another thing: suppose I point a laser beam to the space and by chance this laser beam never finds any kind of matter in its way, where is this laser going to? To an infinite void? Is it correct to say that stars radiate energy to the infinite then?

pigpang
5 replies
9h40m

It's just an interpretation. Your interpretation is similar to the Big Bang model of visible Universe expansion. If you can convince us that your model is better than other models, then we will use your model, but nobody can prove than a model is correct, unless we will find a hidden recorder somewhere which was turned on for few dozens of billion years.

Photon will hit something, or will travel until it will be redshifted to obvilion, or will travel until end of the medium (photon is a wave, so it waves something).

denton-scratch
2 replies
8h14m

(photon is a wave, so it waves something)

That sounds suspiciously like postulating the 'ether'. Surely what a photon 'waves' is the electromagnetic field, which is not a medium, and which fills the whole of spacetime. There is no 'end of the medium'.

pigpang
1 replies
8h1m

"Field" means 3d array of numbers. Spacetime means 4d array of numbers. You are talking about mathematical model of Universe, while I'm talking about physics. Mountain is not just an excitement in a height field. If photon is not waving something, then it's not a wave. Physicists prove that photon is a wave.

michaelsbradley
0 replies
1h49m

If photon is not waving something, then it's not a wave.

No, you’ve simply hit the limits of needing/wanting to understand something in terms of something else similar or more familiar.

Makes the same point on a related matter: https://youtu.be/Q1lL-hXO27Q

pdonis
1 replies
2h45m

> It's just an interpretation

No, it's not, it's our best current model's description of the actual physical reality of our universe.

> nobody can prove than a model is correct

That's true, but it's also true that we can show models to be incorrect, as in, falsified by the data. For example:

> Photon will hit something, or will travel until it will be redshifted to obvilion, or will travel until end of the medium

For the scenario that was posed, a laser beam that never hits anything, none of your statements here are true. The first is ruled out by the scenario; the second is known to be false because there is no "gravitational redshift" of light in the universe as a whole (because models in which there would be such a redshift are known not to correctly model our data on the universe as a whole), and there is no "end of the medium" (again, models in which there would be an "end of the medium", i.e., where the universe stopped containing matter and started being just vacuum, are known not to correctly model our data).

I have described what actually happens in my own response to the GP upthread.

> (photon is a wave, so it waves something).

Light is an electromagnetic wave; what "waves" is the electromagnetic field. (If you use a "photon" model, you are using the quantum electromagnetic field as opposed to its classical approximation.) There doesn't have to be any other "medium"; the electromagnetic field is present everywhere.

pigpang
0 replies
1h15m

No, it's not, it's our best current model's description of the actual physical reality of our universe.

OK, it's our best model, but it doesn't invalidate other models, less complete or less popular, it compete with them.

That's true, but it's also true that we can show models to be incorrect, as in, falsified by the data.

Yep. The article is about the Huble Tension, which invalidates Big Bang model. We still use it.

the second is known to be false because there is no "gravitational redshift" of light in the universe as a whole (because models in which there would be such a redshift are known not to correctly model our data on the universe as a whole)

The Big Bang model is incomplete too: galaxies with FTL speeds, different speeds of expansion, no center of bang, no flows, no source of energy, it stretches time and space, etc.

I assume that the only infinite thing in infinite Universe is Universe itselft. All other things are finite. Thus, a photon has finite life, like any other wave.

there is no "end of the medium"

The right-hand rule in EM suggests that we are in north hemisphere of something, so south hemisphere will have symmetrical rule, unless you believe that God chose right-hand rule for the whole infinite universe. If we are in a sphere, then that sphere rotates and have a boundary.

what "waves" is the electromagnetic field.

"Field" is an array of numbers. You are mixing model and reality.

pdonis
0 replies
2h48m

> before the big bang the whole observable universe was contained in a small sphere and then it started to expand

We have no evidence of any time when the universe was not expanding. At the earliest times we have evidence of, the universe was already expanding (extremely rapidly--much, much, much more rapidly than it is now). At those times, our observable universe was indeed contained in a very small volume.

> suppose I point a laser beam to the space and by chance this laser beam never finds any kind of matter in its way, where is this laser going to?

Since the universe is spatially infinite in our best current model, the laser beam will just keep on going forever.

> To an infinite void?

According to our best current model, no, the laser beam will never stop passing by matter, of approximately the same average density as the matter we can see.

> Is it correct to say that stars radiate energy to the infinite then?

Yes, as long as you recognize that "the infinite" never becomes a "void".

alexb23
0 replies
15h59m

I find helpful this analogy of the space-time (4-dimension) expansion from the big bang: the surface (2D) of an expanding bubble. YMMV.

EnterpriseTell
11 replies
16h55m

In short, even if galaxies weren't accelerating

-Galaxies are not accelerating, space is expanding.

Imagine there are some objects in space all moving in random directions and speeds, relative to Earth...

-No, In your scenario then end result would be a most static average distance between all objects in the universe. As an infinite number of objects come from infinite distances, there would ALWAYS be objects in the neighborhood.

I think what you're imagining is a bunch of objects in a box, give them random vector and then remove the box. If they maintain course, all will eventually move outside the original box boundaries, and away from each other. (not the way the universe is).

They know space is expanding. The primary mechanism we know this is the speed with with we measure an object (moving away) is redshifted. Objects at the same distance from Earth, but opposite regions of space are moving at the ~SAME measured velocity.

There simply is no existing theory which can account for what we are seeing besides space expanding. I'm not big on thinking we understand it all, but in this particular measurement, there is basically zero doubt. Space is expanding, which has the affect of accelerating all objects in the universe away from you, with an acceleration relative to the distance. The more space between you, the more opportunity to expand.

munksbeer
10 replies
8h28m

What does "space is expanding" mean? That the distance between objects is increasing? How can you tell the difference between "space expanding" and "objects moving in a non-expanding space"? Is there any way to tell the difference or is it just that all objects are moving away at the exact speeds that satisfies the "space expanding" explanation and nothing else?

But then, I'm back to what does "space is expanding" mean? What is doing the expansion?

baq
7 replies
7h36m

space expanding means what it literally says: there is more space everywhere at once. there was less a moment ago and now there's more. the longer the distance, the more space gets added in between, thus the effect is extreme on universe scale and undetectable on planetary scales.

munksbeer
6 replies
6h17m

Well yes, but did you read my post in detail? I asked what the difference was between "space expanding" vs "objects moving away from each other".

And secondly, what is the mechanism for "space expanding"? What is "space" in this context? What actually is expanding?

pigpang
2 replies
5h44m

Spacetime is 4D array of points. It's like a movie file, but with 3D frames instead of 2D frames (pictures). Expansion of space means that coordinates of points in a frame changed to move away from us, to match movie (model) with reality.

munksbeer
1 replies
1h18m

An array of 4D points implies some sort of construct upon which the points exist. As far as I know, the that was the concept of "the ether" and that fell out of fashion long ago.

So again, how can you tell the difference between space expanding or two things actually just moving away from each other (to keep it to a very simple example of two bodies).

pigpang
0 replies
42m

Ether now known as "physical vacuum". As I told you already, expansion of spacetime is the mathematical tool, like a shader in OpenGL.

effingwewt
2 replies
3h43m

Let me try to explain- galaxies moving would be like balls floating in a pool. The galaxies are moving across the water.

Space moving would be like balls placed on a bed sheet and the sheet expanding. The galaxies aren't moving- the sheet is.

What's actually happening is more like galaxies moving around on a sheet that's being pulled further at all sides.

munksbeer
1 replies
1h19m

How can you tell the difference?

baq
0 replies
56m

things are moving away from other things the faster the farther away they are, uniformly across the universe. everything is moving away from everything, not just two particular points.

is there a difference if there isn't a difference?

tvshtr
1 replies
4h45m

Imagine a deflated baloon with two dots drawn close to each other. Now inflate it; the dots didn't move but the plane that they were drawn/positioned on did.

munksbeer
0 replies
1h17m

I understand that concept. I'm asking, how can you materially tell the difference?

pdonis
6 replies
18h23m

> How do we know that galaxies are accelerating away from us and not moving at constant speed?

More precisely, we see that galaxies started accelerating away from us a few billion years ago; before that they were decelerating (moving away from us but with the "speed" decreasing instead of increasing).

> People often point to the observation that the further away a galaxy is, the faster it appears to be moving away from us, implying acceleration.

That observation tells us that the universe is expanding, but by itself it does not tell us whether the expansion is accelerating or decelerating or neither. So you are correct that that observation alone is not sufficient to show that the expansion is accelerating.

What we look at to see how the expansion rate changes with time is a comparison of three pieces of observed data, galaxy by galaxy: redshift, brightness, and angular size. The relationship between these three quantities is what cosmologists use to construct a model of the expansion history of the universe, which in turn tells us things like what I said above, that the expansion has been accelerating for the last few billion years but before that it was decelerating.

AlecSchueler
5 replies
17h40m

By few billion are you talking like 3 billion? Why the change?

pdonis
4 replies
17h16m

Meaning, why did the expansion change from decelerating to accelerating a few billion years ago? Because that was when the density of matter, which had dominated the dynamics until then, became smaller than the density of dark energy, which has dominated the dynamics since then. The dark energy density doe not change with time, but the density of matter decreases as the universe expands.

1d22a
3 replies
14h28m

How is is that dark energy density does not change with time? Surely the total amount of dark energy has to be constant (energy can't be created or destroyed, and all that), and then as the universe expands, that's then the same amount of energy over a larger volume, right?

pdonis
0 replies
2h40m

> How is is that dark energy density does not change with time?

Because that's how a cosmological constant works.

There are alternate models where there is "dark energy" (as in, stress-energy that causes accelerated expansion) whose density does change with time (for example, a "Big Rip" model in which the dark energy density increases with time), but such models do not match our best current data.

mr_mitm
0 replies
8h13m

Dark energy may be the energy of vacuum itself, that's why it's constant. And no, energy conservation does not apply in this case. There is a good blog article on precisely this question by Sean Carroll: https://www.preposterousuniverse.com/blog/2010/02/22/energy-...

denton-scratch
0 replies
8h22m

/me not a cosmologist.

I think the story is that dark energy is indeed created, in the new emptiness resulting from the expansion of space.

<mumble> I believe it's supposed to be spacetime that expands, not 'space'. But it's beyond me to explain what that even means; as I understand it, spacetime refers to the whole Universe, across all of time. To 'expand' means 'to become larger over time'. But if the thing that's expanding includes time itself, then I'm bewildered.

colechristensen
2 replies
19h22m

If your idea was the case there would always be new things from very far away heading towards us, this is not the case. If the universe is flat and infinite there would be no end in supply of new galaxies with all velocities and you would always have the same mix as the initial mix of velocities. That’s not what we see.

NemoNobody
1 replies
18h59m

I'm so glad "the infinite universe" as an idea is finally falling off. It works great in the Hitchhikers Guide, I love the floopy mattresses and planets that grow screwdrivers but nothing real is infinite.

I didn't even realize how many people held that belief til that article about how the universe isn't as big as we thought

colechristensen
0 replies
18h0m

Huh? Every evidence points to a flat infinite universe. Nothing but speculation points to anything otherwise.

AnimalMuppet
1 replies
18h42m

I'm not an expert, but I think it's like this:

If the universe were expanding uniformly, we would see galaxies moving away from us. The further away they are, the faster they would move. Distance and velocity would have a linear relationship, with the Hubble Constant as the scaling factor.

But what we actually see is that, if we measure precisely enough, galaxies further away are moving faster than that. The conclusion is that the expansion is accelerating.

pdonis
0 replies
17h18m

> galaxies further away are moving faster than that

No, you have it backwards. Accelerating expansion means, roughly speaking, that we see galaxies further away moving away slower than a "uniform" expansion would predict. Remember that we are seeing galaxies further away as they were a longer period of time ago--so "accelerating expansion" means the universe was expanding slower then, when the light was emitted, than it is now.

Actually, though, we don't observe the distance to a galaxy directly. We infer it from other observations. The actual observed quantities are redshift, brightness, and angular size, and the relationship between those three observed quantities is what tells us the expansion history of the universe.

snowwrestler
0 replies
19h22m

After long enough time, all objects will appear to be moving away from Earth, even if they were moving towards it initially. And after long enough time, the objects that move fastest should be farthest away, by the simple definition of speed!

Sure, but to argue that this explains what we observe today, you would need to show that as of today it has been “long enough,” which is its own can of worms to open.

You might say “obviously it has been long enough for full sorting, because we observe a fully sorted data set of speed correlated with distance.” But that would be begging the question.

ijustlovemath
0 replies
17h54m

One thing that is not often mentioned is that this effect only applies outside the local supergroup; within the supergroup gravity overrides the expansion of spacetime and holds us together (for now!)

Steuard
0 replies
19h30m

You're right: the "more distant galaxies are moving away faster" point is just Hubble's original observation of an expanding universe. It's not an argument for cosmic acceleration. (If you see people making that claim, they're probably either speaking carelessly or not experts themselves.)

The conclusion that the expansion is accelerating was a quite recent result: 1990s, I believe. It's based on careful measurements of supernova explosions of a type with computable intrinsic brightness in increasingly distant galaxies, and the exact pattern seen in their apparent redshifts vs. apparent brightness. It was a shocking discovery when it came out, with two separate teams announcing the result pretty much neck and neck. There's also independent and compatible evidence for acceleration from the exact pattern of variations in the temperature of the cosmic microwave background seen at different points in the sky.

tzs
50 replies
20h24m

A little background based on a few articles about this plus my recollection of PBS Space Time videos on this: There are at least two ways to try to figure out the rate of expansion of the universe (which is called the Hubble Constant).

• From variations in the cosmic microwave background (CMB) which are the result of certain conditions in the early universe it is possible to figure out what the expansion rate should be now.

• From looking at very distant galaxies and noting how far away they are and how fast they are receding from us the expansion rate can be calculated.

Theory says that these should give the same expansion rate. When the rates were first found using those two methods they gave different results, but the error bars on both were large enough to overlap. People expected then that further refinement of both methods to decrease the error bars would converge to some common value.

That did not happen. Refinement of the CMB measurements got to 67 +/- 0.5, and refinement of the galaxy distance/speed method got to 73 +/- 1. Those do not overlap.

This non-overlap between the possible ranges given by the two methods is called the Hubble tension, and it is one of the most irksome problems in cosmology.

Possible explanations include:

• Some sort of error in how we measure the variations in the CMB.

• Some sort of error in the distant galaxy distance or speed measurements, which until the James Webb telescope were almost entirely Hubble telescope measurements.

• We're missing something in our understanding of the physics.

These new results add a bunch of data from the James Webb telescope, which observes in different wavelengths than Hubble. These results fit with the Hubble measurements.

They do not resolve the Hubble tension. What they do is remove most doubt that the distant galaxy results involve some sort of Hubble measurement error. I believe cosmologists are pretty confident of the CMB measurements, and so this will be interpreted as telling us that the Hubble tension is not just a problem with our measurements. There is either physics that we got wrong or physics we need to discover.

lagrange77
16 replies
17h42m

Interesting, thanks!

Out of curiosity: Can we actually differentiate between the expansion of space itself and the drifting of objects within it in the same direction?

dmd
9 replies
17h20m

You would have to come up with an explanation for such drifting to be occurring across the entire visible universe in precisely the same way, across billions and billions of light years.

xeonmc
8 replies
16h50m

Just a fringe speculation without any physical basis: maybe the Big Bang emits a 4-dimensional spherical shell of matter at c?

With the direction in which we are traveling at c called “time”, and the other three which we're not moving through called “space”;

diverging rays having spatial velocity relative to each other equal to tan(alpha) i.e. flat projection;

and at any given point on the 4-sphere's 3D surface everything appear to be diverging in 3D because the emission is spherically symmetric;

and trajectories sufficiently paraxial to us fits the small angle approximation but those diverging widely has much more apparent perspective distortion,

with an error ratio of tan(alpha)/sin(alpha) = sec(alpha) = 1/Lorentz_Factor?

dmd
6 replies
16h34m

without any physical basis

Exactly - the "universe is expanding" explanation has the benefit of having tons of physical evidence for it, whereas your explanation has the drawback of being meaningless not-even-wrong word salad.

dvsfish
3 replies
14h46m

While its probably totally wrong, I think your tone and general attitude to people throwing around imaginative and creative ideas is extremely unproductive and frankly rude. If you want to stifle creative discussions, this is exactly how you'd do it. I found the post to be very stimulating regardless of its validity. And it's not like they're trying to spread it as gospel. There was a lot of disclaimer.

mrguyorama
2 replies
3h58m

If you want creativity, take an art class. This is cosmology, ostensibly a science. Pulling wild ideas out of thin air and basically ad-libbing a "theory" is not.

dvsfish
1 replies
2h56m

So we have to put everyone who dares to imagine down about it? I see value in sharing a wild idea purely for the inspiration it may give others to think about things from a unique perspective. A lot of great science started as a strange idea.

mrguyorama
0 replies
25m

What's the value in """daring to imagine""" about cosmology? This is nothing more than reality based fan fiction.

A lot of great science started as a strange idea.

Disagree. A lot of great science started with "This weird math I'm doing is the only way to fit this data", which is an extremely different thing.

Do you not think physicists and cosmologists and their nerdy friends have thought of wild ideas? Do you not think undergrad students spend drunken friday nights theorycrafting universes?

It's pretty disrespectful to "why don't you just" a profession without showing that you at least did your homework first, or that you have looked at whoever definitely asked that question before you.

There are people in this post reinventing Tired light and Aether based theories for crying out loud, theories with fairly conclusive reasons we abandoned them.

When you just ass-pull theorycrafting without any of the fundamentals, all you do is talk in circles. It's no better than all the dumb "the brain is like a computer" arguments by analogy in neuroscience posts.

It's not even wrong.

We live in a world where the people who had to cheat off me in high school biology angrily insist that the vaccine doesn't work. Do we really benefit from playing such anti-scientific games?

root_axis
1 replies
14h35m

I think if you're going to dismiss the comment in such a flippant and condescending manner you should at least explain what's wrong with it.

dmd
0 replies
6h37m

How does what they said differ from simply saying the universe is expanding - but in a word-salady way that obfuscates it?

jabits
0 replies
9h47m

This is a really interesting thought. I will be noodling on this tonight. Thanks

arandomusername
4 replies
14h32m

I think we observed some of the galaxies moving away from us at rates faster than speed of light, multiple times faster, which wouldn't work with objects drifting away if we hold by that speed of light is max speed matter can move at.

brabel
1 replies
8h10m

That's physically impossible. You can't observe something that's beyond your "light cone", as any galaxy "moving" (it's not really moving, it's the space that's getting expanding) faster than light would be. What you're referring to is the fact that we can confidently predict that galaxies at the edge of the observable universe, which we currently see moving away from us really fast, but as they were in the distant past due to their light taking billions of years to arrive at us, are currently, if we could actually see them where they are right now (again: we can't), "moving" faster than light away from us.

Once a galaxy has moved beyond or "light cone", it's lost forever: you won't see it again even if you try moving towards it at light speed for all eternity.

cygx
0 replies
7h11m

The Hubble sphere (the place where recession velocities hit the speed of light) is not the same as the particle horizon (our past lightcone at current cosmological time, the boundary of the observable universe) or the cosmic event horizon (our past lightcone at infinite cosmological time, the boundary of the asymptotically observable universe).

Cf the last paragraph of https://en.wikipedia.org/wiki/Hubble_volume

Observations indicate that the expansion of the universe is accelerating, and the Hubble constant is thought to be decreasing. Thus, sources of light outside the Hubble horizon but inside the cosmological event horizon can eventually reach us. A fairly counter-intuitive result is that photons we observe from the first ~5 billion years of the universe come from regions that are, and always have been, receding from us at superluminal speeds.

cygx
0 replies
7h15m

This is correct: For example, at time of emission of the light we receive today, GN-z11 had a recession velocity above 4c. A redshift of 1090 (which is the approximate redshift of the cosmic microwave background) corresponds to a recession velocity on the oder of 60c.

BunsanSpace
0 replies
1h6m

The speed of light, or C, is the max speed information can move through our 3d space.

Having objects moving away from us at a speed greater than C, isn't weird. The observable universe is a 3d subspace of a higher dimensional object. A good analogy is a balloon, where there's a 2d subspace on a 3d object that's being inflated. Even if you can only move at a certain velocity, the balloon can inflate such that the 2d surface expands faster than the max velocity we prescribe for it, and would grow faster at the beginning even if we pumped a constant amount of air into said balloon.

Great analogy for gravity too! because you could create dimples in the balloon (gravity wells) which would curve a straight trajectory, while being unnoticed to an observer on the 2d subspace.

mr_mitm
0 replies
8h38m

Yes, the "drifting" component you speak of is called the peculiar velocity. It's responsible for the "finger of god" effect if we don't account for it (galaxies appear to be aligned along the line of sight of the observer) and becomes negligible at large enough distances. I guess technically it's hard to differentiate because it's usually random, but it enters the error analysis and is accounted for.

throwawaymaths
13 replies
18h50m

There's a 'fringe' theory that the CMB is not the echo of the big bang but rather the redshifted black body radiation of intergalactic dust (apparently the numbers are about right). Although the primary proponent of this theory is trying to justify some sort of cyclic universe model with it, if my understanding is correct it would still be compatible with a 'standard' big bang that has the "longer" (aka galactic expansion) timeframe.

And in any case physics still needs to explain what happened to that blackbody radiation.

theoreticalmal
6 replies
16h4m

I can’t believe it. First they take Pluto from us, next they’re going to tell me the static on my tv set isn’t the remnants of the Big Bang, but just hot dust? My childhood is crumbling!

orlp
3 replies
10h46m

Your comment made me realize how some younger people have no idea of, or at least no first-hand experience of static on TV and radio since it is all digital nowadays.

brabel
0 replies
8h39m

I have a super modern flatscreen. Sometimes, when it can't connect to the laptop it shows what I believe is "simulated" static noise! I love that :D

LilBytes
0 replies
10h11m

My great grandparents had a black and white TV when I was a toddler, and I'm 36!

The amount of change millennials and the generations before and after us have seen is boggling.

I was out in country Victoria, Australia a few months back and the internet was TERRIBLE. I'm talking, JPG's loading line by line terrible. And this was on 'alleged' 4G.

I felt pretty nostalgic for all of 2-3 minutes before I started pulling my hair out. I feel nostalgic about it again now.

taneq
0 replies
13h54m

The hot dust is also remnants of the Big Bang, if that helps. Of course, so are we. :)

sebzim4500
0 replies
2h56m

The static on your TV set is <1% CMB, mostly it is interference from other man-made sources and IIRC there is some coming from the sun.

sebzim4500
5 replies
18h40m

How could that be true?

Wouldn't that smear the spectrum so that it no longer so matches the black body radiation of a single object of a set temperature?

throwawaymaths
4 replies
17h48m

No. According to the standard big bang theory the CMB should look like black body radiation.

As for smearing, IIRC black body curve has the central limit property

sebzim4500
3 replies
6h29m

According to the standard big bang theory the CMB should look like black body radiation.

Yes, that's my point.

As for smearing, IIRC black body curve has the central limit property

Surely that can only be true for a very specific distribution of temperatures? To go to an extreme, if you combine the spectrum of two objects of different temperatures you do not get anything like a black body curve.

raattgift
1 replies
4h28m

I agree with you; you can't build up a perfect blackbody spectrum by superposing different near-blackbody spectra, and that led to the cosmological thermalization problem. Furthermore, baryonic intergalactic dust isn't like dark matter: we see redshifted spectral features of components of such dusts imprinted on all sorts of backgrounds from quasars to the CMB. The dusts tend to be greybodies. (I'm ignoring the warm-hot intergalactic medium here).

The CMB has narrow spectral lines imposed on it by galaxies, dust clouds, and similar objects. Those lines can be used to measure the CMB effective temperature at an object's redshift. A specific example of how this works is linked at the bottom of this comment. If a similar highly dusty object is found in the foreground of a bright quasar (merely nearby quasars were used in the case below), we could get an even tighter measurement of the CMB effective temperature at the dust, and perhaps look in new ways at how the CMB deviated (at the dusty object) very slightly from a blackbody spectrum. This amounts to a new line of evidence in the study of the expansion history.

More generally, constraining the evolution of the CMB temperature in the matter-dominated era (T_cmb > 4 K, z > ~ 0.4) will prove fatal to ideas like some in this thread (which is already in trouble because of other spectroscopy e.g. the Lyman-alpha forest and Gunn-Peterson trough).

I don't really understand the summary of the "'fringe' theory" at the top of the thread (and I can't identify which specific theory throwawaymaths means). I guess the idea could be that some unknown mechanism might homogenize the emissive components of baryonic intergalactic dust but not circumgalactic medium spectral features. I suppose one could try to investigate a non-adiabatic expansion to explain those features found in the CMB in a nonstandard way. Non-adiabatic components in the expansion have been examined by various theoretical teams in the context of quantum cosmology and/or "fifth force" dark energy, although mostly at much larger lookback times than the the standard surface of last scattering.

I don't understand what was meant by '"longer" (aka galactic expansion) timeframe'.

Finally, "echo of the big bang" isn't really helpful in understanding the origin of the CMB photons, why they're now filling space with a thermal Planck spectrum, and why we detect small variations in that spectrum in narrow views along different directions, but like "fabric of spacetime" I guess we're kinda stuck with the expression.

- --

(Open Access) Riechers, D.A., Weiss, A., Walter, F. et al. Microwave background temperature at a redshift of 6.34 from H2O absorption. Nature 602, 58–62 (2022). https://doi.org/10.1038/s41586-021-04294-5

throwawaymaths
0 replies
1h23m

I am not a physicist, but iirc the idea of the alternative explanation of the CMB is that if you time evolve backwards intergalactic dust over expansion, you reach a point where the dust gets really dense and effectively opaque.

So this isn't summing all of the dust you go through, but rather the redshift all the way back to an era when the universe was super dusty (with the expected thermal signature of that era)

throwawaymaths
0 replies
1h28m

if you combine the spectrum of two objects of different temperatures you do not get anything like a black body curve.

Also true for two normally distributed variables and yet the normal distribution has the central limit property.

Presuming the hypothesis, do you suspect that you'd be looking at exactly two grains of dust, one of which has a crazy high temperature and the other has a crazy low temperature in any given pixel of the CMB?

rf15
11 replies
19h47m

This, together with the endless philosophising around dark energy, dark matter and whatnot paints a pretty strong arrow towards our models having some flaws when it comes to their large-scale application. I hope to live long enough to see where we made our mistake and get a better model.

pishpash
4 replies
19h35m

Almost every model we build of more trivial things based on observation always turns out to be not really right. I cannot imagine why one of the universe that has had multiple version updates in the last 100 years to not also be grossly mistaken. I also don't expect the full model to be simple or beautiful. We may be thinking wishfully based on massive extrapolation and cutting corners to suit our narrow view into the world.

CuriouslyC
2 replies
18h44m

I think the truth is likely that we live in a very complex universe that can be approximated by these sweeping laws in general, but is very "messy" close up. Consider that while the universe roughly obeys probabilities, it also has will and intent (to a degree we can debate in another conversation).

kaashif
1 replies
15h59m

What do you mean by will and intent?

I guess if humans have will and are part of the universe, then the universe has it too, but I don't think that's what you meant.

CuriouslyC
0 replies
5h9m

Hopefully we can agree that at least part of the universe "chooses" outcomes, rather than them occurring randomly. You could assume that "choosing" is what random outcomes feel like but I don't think that's productive.

Given that we have evidence of physical systems evolving their states by conscious choice, that begs the question "Why would large, complex physical systems have a governing force that doesn't exist in smaller physical systems?" Occam's Razor suggests that consciousness and will drive the evolution of all physical systems, and our elegant equations are merely statistical approximations following the law of large numbers given a population with a natural variance. Of course we can debate what it means for a low information system to be conscious and have will, but I think that's a more meaningful debate than trying to pinpoint the exact moment that a system becomes complex enough for the magical emergence fairy to sprinkle consciousness dust on inert matter.

qup
0 replies
11h4m

It's probably spaghetti code

dclowd9901
1 replies
18h10m

Interesting to think we only got the “middle” size physics right, but we can’t reconcile it with micro or universal physics.

ethbr1
0 replies
17h4m

I think about it as getting practical / low-tech observable physics right, then expanding out from there.

< atomic and > planetary have very important applications!

But I'd argue not nearly as many as "Here are all the formula that govern a falling apple."

hinkley
1 replies
19h13m

I still have $20 down on "We are living in a universe inside a black hole."

hughesjj
0 replies
18h41m

I mean effectively we are regardless. The cosmological event horizon is a thing.

Also, I forget off the top of my head, but there's some oddities depending on your chosen frame of reference when looking @ hawking radiation (skip to part about relativity of the vacuum) that I think could apply to the cosmic event horizon as well

https://youtu.be/isezfMo8kWQ?si=5m_L6JtZ7p7Ls6xH

superjan
0 replies
12h3m

And don’t forget cosmic inflation.

xeonmc
2 replies
18h53m

Could it be that nearby galaxies are akin to small angle approximations in spacetime trajectory, but as you get really far away (e.g. CMB) the perspective distortion increases hyperbolically? I notice that if you normalize the Hubble data by their Lorentz factor you get back a constant expansion rate: https://www.desmos.com/calculator/llhnja1ocb

rhelz
0 replies
18h1m

For real? That is a very interesting observation.

Kerbonut
1 replies
14h53m

Could it be the rate of expansion is not constant?

michaelsbradley
0 replies
19h24m

"Humason assembled spectra of the nebulae and I attempted to estimate distances." So wrote Hubble of his colleague Milton Humason in 1935 by which time spectra had been obtained for over 150 nebulae. Hubble was a stern warner of using the Doppler effect for galaxies and argued against the recessional velocity interpretation of redshift, convincing Robert Millikan, 1923 recipient of the Nobel Prize for Physics and director of physics at the California Insitute of Technology, that the redshift interpretation as an expanison of the universe was probably wrong, the year before both of their deaths in 1953.

Hubble ended his book Observational Approach to Cosmology[+] with the statement:..."if the recession factor is dropped, if redshifts are not primarily velocity-shifts, the picure is simple and plausible. There is no evidence of expansion and no restriction of time-scale, no trace of spatial curvature, and no limitation of spatial dimensions. Moreover, there is no problem of internebular material. The observable region is thoroughly homogeneous; it is too small a sample to indicate the nature of the universe at large. The univers might even be an expanding model, provide the rate of expansion, which pure theory does not specify, in inappreciable. For that matter, the universe might even be contracting."

[+] https://ned.ipac.caltech.edu/level5/Sept04/Hubble/paper.pdf

source: https://plasmauniverse.info/people/contributors.html

JohnMakin
38 replies
21h13m

This sort of brushes on it but for a long time there was hope at resolving the Hubble Tension by saying that Hubble telescope’s measurements were incorrect, because that would be the most simple explanation. This was not the case, so if anything, the mystery deepens. I don’t know for certain but I believe Hubble’s estimation has been widely accepted for a while though, because we’ve been using the 13.8 billion cosmological age estimate ever since I started brushing up my layman's understanding of the subject.

orra
36 replies
20h45m

we’ve been using the 13.8 billion cosmological age estimate ever since I started brushing up my layman's understanding of the subject.

I remembered the age of the universe as as 13.7 billion years, but I wasn't sure why that was.

Well, the initial WMAP results in 2003 supported an age of 13.7 billion years. Later results nudged this upwards to 13.8 billion years. Of course, all the results have error bars.

explaininjs
35 replies
20h26m

Of course, all the results have error bars.

Not to mention the underlying philosophical assumptions, such that the "rate of time" has been constant across all of... time.

Aka: How do we know a "year" 13 Billion "years" ago bears any resemblance to one now? What would it mean for it not to?

Veserv
12 replies
19h22m

Well for one, if the laws of physics are not time invariant (i.e. the laws of physics are not the same at all points in time) then energy can be created or destroyed [1]. So that would be quite a shocker.

[1] https://en.m.wikipedia.org/wiki/Noether%27s_theorem

explaininjs
6 replies
17h19m

Do you think the current amount of energy in the universe is 0? If not, how was it created?

baq
5 replies
9h40m

That's a completely different question, to which the answer is 'we don't know'.

explaininjs
4 replies
6h38m

It's really not. If there is nonzero energy, and our current models don't allow for energy to be created, either we need new models that do allow for energy to be created (time-variant physics, for instance), or we need the existing models to propose a way it was initialized. Without that there's a gaping hole in the model the size of all of the energy in the universe.

Certainly you can't 1) have a model where energy is constant 2) believe there is nonzero energy in the universe and 3) dismiss any model where energy can change as bogus out of hand because you believe it should be constant, without counter-proposing how it even got here.

mrguyorama
3 replies
3h52m

or we need the existing models to propose a way it was initialized.

We have that, it's the big bang.

explaininjs
2 replies
3h31m

How remarkably unsatisfactory.

The argument being:

You can't propose physics ever changed, that'd mean energy could have been created at some point!

Energy was created at some point, wouldn't it be nice to know when?

Everything created at the Big Bang exactly as it is now, stop asking questions.

orra
0 replies
2h34m

You're asking to go too far back in time.

The difficulty is we don't know what happened in the first planck second after the big bang, let alone before the big bang (if that's meaningful).

We haven't unified quantum mechanics and relativity. Hence we can't be certain that singularities exist, or that the universe started off in a singularity.

baq
0 replies
2h48m

Nobody argues it wouldn't be nice. Physics argues we can't ever know. It's a rather significant difference and if this gets proven wrong, you'll find that physicists are the ones partying the hardest.

itishappy
2 replies
18h10m

Doesn't dark energy do exactly that? 70% of the energy of the universe doesn't seem to want to play by our rules!

digging
1 replies
17h3m

Yes, but my understanding is that dark energy doesn't play by the same rules, it's an exception. I certainly can't explain why but also it may not be known exactly why, given dark energy is an unexplained phenomenon.

cygx
0 replies
7h35m

The party line is that energy is not conserved at cosmological scales. However, it's more of a semantic question: We can tell you exactly by how much it gets violated (that's basically the first Friedmann equation), and if you prefer, you can attribute the missing energy to the gravitational field. A lot of physicists don't like that approach as it isn't possible to write down a corresponding stress-energy tensor, ie gravitational energy cannot be properly localized.

hnfong
1 replies
12h35m

Saying "X is true because otherwise, it would be quite a shocker" isn't really a proof.

It's almost an appeal to common sense. (which, admittedly, is often the best argument we have). IMHO there's often a bit of over-confidence among scientists about the universe being 13 billion years old and what happened during the Big Bang, if it just relies on such a common sense argument.

I know it's unscientific to suggest maybe laws of physics are not time invariant across such scales, because until we have a time machine we can't test this theory, but then flipping the argument (that laws of physics are definitely time invariant) is also technically unscientific -- we're basically assuming this without strong evidence.

This goes back to the old debate on the problem of induction in science, (see David Hume, Karl Popper, etc.) and I think it isn't emphasized enough in modern discussions that, perhaps, there's a small chance that these foundational concepts in physics could be invalid.

Veserv
0 replies
11h33m

I did not present proof. They asked what it would mean and a consequence of physical laws not being time invariant would be that energy does not need to be conserved.

To the extent that relates to epistemology that would be more like anti-proof I guess? It does not prove that the laws are time invariant, rather it raises the bar to demonstrate that the laws are not time invariant because that means energy need not be conserved. You can not get one without the other without attacking even deeper fundamentals of modern scientific models. So you must either demonstrate the linked claims, which are pretty foundational themselves, or you must overturn basically everything; both of which demand very robust evidence.

dartos
6 replies
20h21m

I’d assume that we have some notion of how the laws of physics have changed, if at all, since the Big Bang.

We measure time in vibrations of a cesium isotope IIRC

db48x
2 replies
19h48m

No, we measure the frequency of vibrations of _light_, not of a type of atom. Specifically, it is light emitted by cesium atoms that are transitioning from one specific energy state to another specific energy state. Although this is arbitrary, it is highly reproducible and would give precisely the same measured lengths of time at any point since the big bang.

elashri
1 replies
19h27m

That assumes that fundamental laws of physics did not change (will not change). This is what we believe and have no evidence otherwise. This is important since we rely on measuring atomic transitions of cesium atoms which itself were formed/forming billions of years after the big bang itself.

The laws of physics invariance under time is a core to our understanding. It would be very disrupting if we found otherwise.

not2b
0 replies
18h27m

Right, but most deviations one can think of (like, changes over time to physical "constants") would have an observable effect, so ancient galaxies would look much more different from modern galaxies and spectra wouldn't look the same other than a red shift, which moves all the lines in a uniform way.

drjasonharrison
0 replies
18h46m

And we can split hairs and conclude that for "the last two billion years, on this planet, in this galaxy, the physics affecting nuclear decay have not changed"

It's great to know that say dating using carbon-14 decay is still useful over those time ranges on planet earth (I don't know if that is something we care about given that fossils don't tend to contain much carbon, but coal and oil deposits are around 400 million years old).

I don't want to imply that this is too small of a sample size, but I will imply that nuclear decay, and the movement of galaxies across the universe might be unrelated. Don't know. Not sure how we'd measure that. Supernova observations would tell us about nuclear fusion and it's limits. Does it tell us about nuclear fission? I don't know.

mr_toad
0 replies
5h49m

“Before” the Big Bang it’s not even clear that there were laws of physics.

dwattttt
4 replies
19h54m

How do we know <anything>? Observations we can make plus models that relate those to the things we can't observe directly.

vlovich123
3 replies
19h18m

Unfortunately we can’t observe fundamental things like “what are the rules of physics and time at the beginning of the universe?”. We look for clues and make large assumptions, but given that the universe experienced a 10^78 factor expansion during the Big Bang, assuming that actually happened, then why would it make sense to assume that the rules of the universe today are the rules for the very early beginning of the universe? A strand of DNA would become 10 light years. Given that relativity redefined our understanding of basic physics but only applies as we approach the speed of light, it would stand to reason that the rules of physics would be different from our current models based on today’s observations when the matter of the universe is packed much more tightly together.

itishappy
2 replies
17h43m

Entirely reasonable assumptions! Our models match surprisingly well though... The CMB has a blackbody spectrum that aligns with predictions, we see galaxies more or less when and where we'd expect them, stellar populations look like what we'd expect for a universe made of hot hydrogen, and more! It's not quite perfect, but modern physics explains stuff really quite well even billions of years ago!

vlovich123
0 replies
15h0m

The inflationary epoch where it expanded by 10^78 in volume happened in the first 10^-32 seconds. The furthest galaxy we can see (fairly poorly) is 300M years after the Big Bang. It's likely if time or the rules were different, 300M years was enough for things to mostly die down to steady state. And as you say, they match more or less but those errors could easily hide remnants of when things were different. Of course, these are all numbers that assume the Big Bang theory is correct which is difficult to impossible to falsify since we can't possibly observe or test anything from that long ago. We'll have to wait to see if refinements to our model that clear up contradictions change what we think about the beginning of the universe and other boundary conditions.

explaininjs
0 replies
17h12m

Modern physics is guided by those observations, they can't be then used as an argument for its veracity.

Let's say I walk into a room and observe someone writing a tally mark on a chalk board once every second. I count 4x10^17 tally marks. I might assume that 4x10^17 seconds ago that same person entered the room and started tallying. I might even observe for the next 4x10^17 seconds they continue to tally. Heck I might even see a recorder going that when I play back at what I assume to be 1x speed, has chalk scratches at regular intervals for 4x10^17 seconds. I still don't have any actual evidence that they started those 8x10^17 seconds ago.

wavefunction
3 replies
18h50m

I have had the same thought that primordial reality had the same timestream as us but it was much longer, like the first "year" of reality was far longer than one year today, just the thinking "could time have been shorter or longer, why not"

denton-scratch
2 replies
8h37m

the first "year" of reality was far longer than one year today

This talk about the "rate of time" doesn't make any sense to me. A second takes one second, and always has.

Isn't this like asking whether the length of a metre might have changed over time? Or the mass of a kilogramme? It looks to me like a category error.

explaininjs
1 replies
3h26m

`This talk about the "rate of execution" doesn't make any sense to me. A cycle takes one cycle, and always has.`

The root of the question is whether there exists an external "system clock", and what that would even mean.

denton-scratch
0 replies
46m

whether there exists an external "system clock"

I'll ignore your analogy with a CPU, which I don't think is apposite.

Whether there's a 'system clock', a sort of reference clock that can tell you how fast time is passing, so you can calibrate other clocks against it, seems to be the same category error. As far as I can see, either time is all there, all at once; or one second lasts exactly one second. (I've perpetrated the same category error there, because a second doesn't 'last' for some period of time; it just is).

[Edit] That's not very clear. I mean: if you are measuring a distance, you use a calibrated ruler. If you doubt the calibration of your ruler, you might recalibrate for precision. But you don't have a clear idea of what one foot is, it doesn't make any sense to ask whether your one-foot ruler has grown or shrunk; how would you tell if the purported 'fact' is true or false?

And if some fact about the Universe means that it has grown or shrunk, how would you tell? If you can't in principle tell whether a 'fact' is true or false, it follows that the 'fact' isn't a fact, because it has no effect on anything.

See Russells Teapot.

fooker
3 replies
20h22m

Philosophical indeed, as it's impossible to define the idea of a rate of time, when the idea of rate is defined in terms on time itself.

explaininjs
2 replies
13h19m

Yes, an external reference clock is needed.

If a computer chugs along doing { counter++; } at 1 clock cycle per clock cycle for 13.7 billion clock cycles, it will think 13.7 billion clock cycles have passed when counter is 13.7 billion.

On the other hand, if a computer chugs along at one clock cycle per clock cycle for 1 clock cycle, and reads &counter and sees 13.7 billion, it will think 13.7 billion clock cycles have passed.

Either way it's perfectly capable of introspecting it's source code and logically stepping back until the memory location was 0 to see how many clock cycles would have been required to reach it's current state, but that sort of reasoning is completely devoid of meaning without both perfect knowledge of what the true start state was, and a guarantee that no external influences have occurred.

Here in reality, we know neither the our start state nor our isolation level, but the hubris of many is too string to not at least try finding some logical step-back functions and iterating them until they don't know how to go any further, then proudly proclaiming that "The Start". (after all - how could it not be The Start, look, I can iterate the inverse of my step-backward function from then to now and it matches! QED!)

jMyles
1 replies
12h32m

I agree with you 100% on your assessment of what can be known, but I think I disagree nearly as strongly on your assessment of the humanity that springs from attempting to know it.

To exist in the state you describe - with neither start state nor isolation level measurable in any tamper-proof way - and to yet still dedicate one's life to observing and pondering the complexity of the resulting cosmos is, to my eye, laudable and beautiful.

Where you see hubris, I see humility. Everyone who attempts to expand the corpus of human understanding of cosmology knows that the endgame is somewhere short of perfection. And yet they are inspired to carry on. It seems to me that matters of state, economics, medicine, technology, and many other fields will benefit from a similar disposition.

explaininjs
0 replies
3h23m

I think the hubris comes in when folks assume that because that answer is presented by Science/Observation rather than Religion/Philosophy, it's somehow guaranteed to be "more accurate", when in reality both are guesses with equally unknowable error bars.

speak_plainly
0 replies
20h9m

Time in this context is just an arbitrary measurement. Like extrapolating the calendar back to the Big Bang which is when space/time began, another way to think about time.

mr_toad
0 replies
5h53m

Rates are measured with time as the divisor. What does rate of time even mean?

physicles
25 replies
13h34m

The article mentions the cosmic distance ladder, which is one of my favorite things in all of science. How do we know how far away the really far stuff is? It's non-trivial and I find the history fascinating.

It all started with knowing the distance from the earth to the sun. Nobody had a clue until Richer and Cassini got within 10% in 1672. Then we nailed it down in 1769 with James Cook's voyage to Tahiti, the primary purpose of which was to observe the transit of Venus from the other side of the world.

From there if you know basic geometry, you can observe the nearby stars shift a bit when the earth goes around the sun (parallax), but that only works to about 10k light years.

Then, we discovered a couple unbelievably convenient astrophysics hacks: Cepheid variables (Henrietta Swan Leavitt, 1908) and Type 1A supernovae (Subrahmanyan Chandrasekhar, 1935, the namesake of the Chandra X-Ray Observatory). These allowed us to move out a couple more rungs on the ladder.

From there, the relationship between redshift and distance becomes significant and that takes us to the edge.

https://www.uwa.edu.au/science/-/media/Faculties/Science/Doc...

luc4sdreyer
7 replies
8h58m

Nobody had a clue

In the 3rd century BC, Aristarchus calculated that the Sun was between 18 and 20 times farther away from the Earth than the Moon, and proposed the Heliocentric model as a result. The true value is instead approximately 400 times. But it's incredible given that he didn't have lenses, the value of Pi, and that the Geocentric model was considered correct until 1800 years after his death.

https://en.wikipedia.org/wiki/Aristarchus_of_Samos#Distance_...

Nice video about the cosmic distance ladder by Terence Tao: https://www.youtube.com/watch?v=7ne0GArfeMs

whimsicalism
6 replies
7h34m

yeah i would call estimating it as 20x when it is actually 400x firmly within “not having a clue”.

He didn’t say nobody had a clue about heliocentrism.

dexterdog
3 replies
7h7m

But I believe he was the winner for many years under price is right rules.

justsid
1 replies
2h1m

Can you claim to be a winner under the rules of a game that won’t be invented yet for hundreds of years?

nashashmi
0 replies
1h41m

Yes. The fact that even this was a thought and he was charting the space bodies and trying to establish distances between them. The rules were invented. The players just were not famous.

nsxwolf
0 replies
2h36m

Smugly: "1 mile."

cwmma
0 replies
6h40m

Plus they didn't know the distance to the moon

Cthulhu_
0 replies
46m

I appreciate that he worked on it at least. Around the same era, someone else calculated the circumference of the earth (and that it was round) in a pretty accurate fashion (between −2.4% and +0.8% off) based on measuring shadows on equally sized posts at different locations on the same date. Googled, it was Eratosthenes, the cities were Alexandria and Syene/Assuan.

rexer
6 replies
12h5m

Great comment! Maybe you can help me with a book recommendation?

I was recently looking for a book which was basically your comment, but more in depth and covered the last couple thousand years. I wanted a to read about the history of astronomy - yknow, what was the state of the art in, say, 1350 or whatever. If you know of anything, I’d be super interested!

HarVard93
0 replies
6h50m

ya

zuzun
0 replies
8h54m

Big Bang: The Origin of the Universe by Simon Singh.

severila
0 replies
9h0m

You might be interested in "Unrewarded" by "Ben Moore" which has an interesting take by telling the history of astronomy through the lives of those that made these discoveries but were not awarded a Nobel Prize.

physicles
0 replies
11h2m

Unfortunately I don't have any books to recommend. I don't remember where I learned about Cepheid variables and type 1a supernovae (maybe science shows, maybe youtube, ...) but I learned about the transit of Venus stuff on a big Wikipedia rabbit hole one evening.

I think the pre-quantum mechanics era for physics and astronomy is super interesting. People figured out so much with such primitive tools, and it's all very accessible and easy to understand.

nolta
0 replies
3h54m

You might like "Coming of Age in the Milky Way".

whimsicalism
0 replies
7h31m

Strongly doubt this

cygx
0 replies
7h47m

Not sure that's convincing: Why would I multiply a unit of time (Yug) with unit of distance (Yojan) to arrive at the distance to the sun? Also note that per Wikipedia, the historical value of the Yogan can range from 3.5km (~2.2 miles) to 15km (~9.3 miles). How was the value of 8 miles chosen?

QuadmasterXLII
0 replies
6h30m

I'm sure your goal is to boost the intellectual reputation of ancient hindu philosophy, but you're mostly scuffing the intellectual reputation of hinduism.stackexchange.com

tails4e
2 replies
9h58m

I've often thought about this myself. I'm sure scientists involved are aware of the compounding errors with each step and build that in, but I'd love to see an analysis that breaks that down. When I first saw it I thought the errors due to cephids must be a large component of uncertainty, but really I've no idea how well contained that is.

sanxiyn
1 replies
7h1m

Error analysis of cosmic distance ladder is fiercely technical subject. https://arxiv.org/abs/1103.2976 Table 5 is titled "H0 Error Budget for Cepheid and SN Ia Distance Ladders". (It is old and the field is moving fast, but this is what I happened to remember.)

Among total error of 3.1%, Cepheid reddening is 1.4% and the second largest source of uncertainty. SN Ia statistics is the largest with 1.9%. Rarely discussed in popular treatment is anchor distance, the third largest source with 1.3%. It is uncertainty of bottom lungs of the ladder, eg the distance to Large Magellanic Cloud before Cepheid and SN Ia are involved.

raattgift
0 replies
3h20m

Rarely discussed in popular treatment is anchor distance

Yes, and I'm sad that it's so rare.

The Megamaser Cosmology Project is incredibly awesome; combining line of sight acceleration, velocity, velocity gradient, and observer angle on the sky is very "anchor".

https://safe.nrao.edu/wiki/bin/view/Main/MegamaserCosmologyP...

Gaia seems to be making good progress on direct parallax up to kiloparsecs.

And always happy to plug ASAS-SN <https://www.astronomy.ohio-state.edu/asassn/index.shtml> (The deleted @SuperASASSN was one of the best astro follows on twitter some years ago), who have found an awful lot of detached eclipsing binaries: <https://academic.oup.com/mnras/article/517/2/2190/6695108?lo...>.

joshjje
0 replies
6h11m

I imagine there must be lots of gravity lensing going on as well, not sure how they deal with that.

Scubabear68
0 replies
8h5m

It is amazing how much of basic science is rooted in simple geometry.

gregorymichael
22 replies
21h6m

What are the implications?

mr_mitm
10 replies
20h59m

Either we live in an unusually under dense region of the universe or our models are wrong ("new physics").

amethyst
9 replies
20h56m

Or it's a simulation and someone keeps pushing changes to production.

ducttapecrown
2 replies
19h52m

All the expert software engineers agree this is the most likely explanation. Have physicists looked into this?

NateEag
1 replies
16h28m

All the expert software engineers agree this is the most likely explanation.

That's quite a strong claim. I'm skeptical. Sources?

Have physicists looked into this?

They shelved it right next to "God Made The Universe" in the "Unfalsifiable Propositions" section, under the title "Grad Students Made The Universe."

kaashif
0 replies
15h52m

I'm reading their comment as a joke about how software engineers tend to overestimate their own expertise on things like physics and are not actually anywhere close to experts.

Software engineers presenting weird pseudo science as serious physics is one way this manifests.

I could be wrong.

cwillu
1 replies
20h47m

Which would also count as new physics.

yreg
0 replies
19h36m

With even more literal meaning of new.

queuebert
0 replies
20h45m

Somewhere aliens are making fun of how shoddy our simulation is coded.

lajr
0 replies
20h46m

I wonder if their introspection is good enough to have our population on a Grafana dashboard somewhere

imzadi
0 replies
19h45m

Someone keeps running gparted on our partition

brcmthrowaway
0 replies
20h24m

It's definitely a simulation at this point

kromem
10 replies
21h0m

Basically it means that we can't regard the Hubble result as a mismeasurement and the age of the universe seems to be different depending on how you measure it.

From the article:

"The bottom line is that the so-called Hubble Tension between what happens in the nearby Universe compared to the early Universe’s expansion remains a nagging puzzle for cosmologists. There may be something woven into the fabric of space that we don’t yet understand."

washadjeffmad
7 replies
20h32m

Sounds like they don't want to spoil everyone's research grants!

jdiff
6 replies
20h14m

Outside of the tinfoil, it just sounds like the universe is complex and not always predictable.

harywilke
3 replies
19h54m

It's amazing that barely a 100 years ago The Great Debate in astronomy was weather the Milky Way was the extent of the universe or things like Andromeda were their own 'island universes'. In the 1920s, Edwin Hubble showed that Andromeda was far outside the Milky Way by measuring Cepheid variable stars. These are the same stars that we are measuring today in this debate. https://en.wikipedia.org/wiki/Great_Debate_(astronomy)

chadcmulligan
2 replies
19h27m

And not much longer before that the discussion was how long would the sun last - 5,000 years or so was the estimate if it was a big ball of burning gas (source: A scientific American article I read, wish I could find it again, hoping someone here knows)

chadcmulligan
0 replies
18h53m

That's the one, thanks, I've been looking for it for ages

washadjeffmad
1 replies
20h0m

I meant it as a comment on the phrasing. Potentially jeopardizing a field's cash flow is a legitimate worry, and I see a few have felt that, as well.

bigbillheck
0 replies
19h56m

There's a whole lot of open problems in cosmology, nobody's going to be out of work if they solve this one.

techwiz137
1 replies
20h12m

I just think it means the expansion rate is not a constant, but a variable.

rihards__
0 replies
13h16m

I kind of agree with this conclusion.

Before we know better it can be just that spacetime was expanding at a different rate (we still would need at least one another Planck that operates in roughly same range to confirm this).

Hubble wavelenght range - 0.1 to 0.8 μm Webb wavelenght range - 0.6 to 28.3 μm Planck wavelenght range - 330 to 10000 μm

My understanding is that Planck was observing photons that have happened much more earlier.

westurner
9 replies
20h42m

Hubble's law: https://en.wikipedia.org/wiki/Hubble%27s_law

Expansion of the universe: https://en.wikipedia.org/wiki/Expansion_of_the_universe :

While objects cannot move faster than light, this limitation only applies with respect to local reference frames and does not limit the recession rates of cosmologically distant objects

Given that v is velocity in the opposite direction, and c is the constant reference frame speed of light; do we account for velocity in determining whether light traveling at c towards earth will ever reach us?

  v - c < 0 if v>c
  v + c > c if v>0
Are tachyons FTL, is there entanglement FTL?

How far away in light years does a mirror in space need to be in order to see dinosaurs that existed say 100 million years ago?

a_random_canuck
4 replies
15h50m

do we account for velocity in determining whether light traveling at c towards earth will ever reach us?

No, because the speed of light is constant for all observers. From our frame of reference on earth, light from distant receding galaxies is always moving towards us at exactly the speed of light, c. Those galaxies also observe the light moving away from them at exactly c.

That seems contradictory and unintuitive, that two observers moving away from each other both measure light moving c relative to themselves, but that’s reality.

It’s another measurement that changes: if c is always constant, then it must be the passage of time and the distance travelled that we observe differently.

pigpang
3 replies
10h40m

That seems contradictory and unintuitive, that two observers moving away from each other both measure light moving c relative to themselves, but that’s reality.

Light is a wave. Photon is a complex thing (Hopfion?), but it's a wave, so it waves something, a medium. Speed of wave propagation in a medium is constant. IMHO, it's intuitive.

mrguyorama
2 replies
3h5m

but it's a wave, so it waves something, a medium.

Which was unobserved in the michleson-morely experiment and other followup experiments because....?

pigpang
1 replies
27m

LIGO/Virgo are better version of Michelson-Morely experiment. Gravitational waves are found, so existence of a medium is confirmed.

mrguyorama
0 replies
20m

Gravitational waves existing doesn't confirm the existence of a medium any more than discovering light behaves like a wave confirms the existence of the aether.

If you want to posit that light has a medium, you need to redo 100 years of physics, so start showing your work. You mention elsewhere "doing a calculation", and that's just not nearly good enough. You want to overturn perfectly working physics, you NEED to show up with receipts. That's table stakes.

arbitrage
2 replies
18h39m

Tachyons aren't a thing. Tachyons are sci-fi nonsense.

Nothing in the universe can travel faster than the speed of light. This does not hold for the universe itself. It can and does expand faster than the speed of light, using specific reference frames (i.e., big enough).

So, space can increase FTL. Particles do not travel faster than light tho, that is nonsense.

rthnbgrredf
0 replies
4h36m

Well, we have (virtual) particles that can travel backwards in time, without breaking causality. There's no proof that Tachyons exist, they are purely hypothetical, but they are not outright nonsense.

cbolton
0 replies
3h32m

Take a star in a region of the universe that recedes from us at 3c. In what sense is the star not traveling faster than the speed of light relative to us?

blackbear_
0 replies
19h38m

do we account for velocity in determining whether light traveling at c towards earth will ever reach us?

As far as I know this is not necessary because the speed of light is constant regardless of the velocity of both the source and the observer (this is Einstein's special relativity: https://en.m.wikipedia.org/wiki/Special_relativity)

mjfl
8 replies
19h41m

It is very fortunate that the universe is expanding. This provides a virtually unlimited source of energy.

digging
7 replies
16h55m

Without being able to harvest dark energy, it's actually the exact opposite.

mjfl
3 replies
15h34m

The extra energy comes from the increase in gravitational potential energy between objects. The accelerating increase in this potential energy implies the total energy of the universe is increasing. I’m not talking about harvesting dark energy directly.

antod
2 replies
14h54m

I'm no physicist, but isn't the gravitational potential energy between two objects inversely proportional to the distance between them?

mjfl
1 replies
14h46m

it is, in the negative direction though. So increasing the distance increases the potential energy by making -1/r closer to zero.

antod
0 replies
13h44m

Heh, I had to go off and read some stuff to get my head around what "negative" potential energy would even mean (high school physics was decades ago).

So... the negative is just a convention to represent work done against the "field", and positive is work done by the field? ie more of a vector than actually being negative energy? I think I get that bit now.

So now I'm wondering if that still applies to an expanding universe vs eg a rocket leaving earth. If things aren't moving further apart by work (ie force x distance) being done against their gravitational fields so much as space time expanding, is there an increase in potential energy? And then if objects are moving apart faster than escape velocity, could that still be seen as increasing potential energy?

I think I'm confusing myself further...

puzzledobserver
2 replies
15h23m

What would a hypothetical dark energy engine look like?

undersuit
0 replies
14h46m

It would harness dark energy. /s

We haven't detected dark matter or dark energy outside of their visible effects on the larger universe. Maybe we can't interact with it with baryonic matter. A dark energy engine made out of dark matter would be invisible.

mjfl
0 replies
14h9m

two black holes in highly elliptical orbit, so they they nearly collide when they come together. at the center of mass is a cloud of iron atoms. When the black holes nearly collide tidal forces across iron nuclei rip them apart. These ripped apart atoms can then be re-harvested and used as fuel in a fusion engine. The energy loss from ripping the atoms apart is then recovered by the black holes by taking advantage of dark energy when they go apart, which increases the distance between them, increasing the gravitational potential energy.

FredPret
5 replies
20h52m

Unbelievable how much we can work out from just the odd photon hitting us from somewhere in the great unknown.

daxfohl
1 replies
20h25m

It's astounding to me that space is so empty that in the billions of light years from here to the edge of the universe, there's orders of magnitude less total interference than what you get from a small cloud.

It's also astounding that celestial objects emit enough photons that thousands per second travel within the arc that goes the distance from the star to somewhere inside the radius of your pupil.

And if this wasn't the case, then we'd never even be aware of any of this.

suzzer99
0 replies
11h25m

Also that the light and gravitational waves from colliding neutron stars can travel 100 million light years and arrive at earth within a second of each other. That's a mind-bogglingly small amount of drag.

CobrastanJorji
1 replies
20h22m

"The odd photon hitting us" is also a pretty good description of eyesight, radio, and fiber optics.

fooker
0 replies
20h21m

Clearly the even photon is for parity checking and error correction ;)

golemotron
0 replies
20h27m

Occam's Razor does a lot of heavy lifting.

jjbinx007
1 replies
19h13m

I wonder if the mods can change the main article link to the one you provided instead?

dang
0 replies
16h33m

Indeed we can!

cwillu
0 replies
20h48m

Added bonus: it has 50% less animated “responsive” design.

kovacs_x
4 replies
4h55m

I'm 1000% sure, in 20, 50, 100, 500 years humans will look back on "science of 20/21th century" as we look now on say 16/17th century- there were some things that were "mostly" right, but most of it was incorrect / inprecise / incomplete and was a fantasy of earlier thought up models.

It's been like that always. Don't think it'll be different this time. Though people of every time thought they had (almost) complete understanding of the universe. I remember reading that at the beginning of 20th century science was largerly considered complete... we know how it turned out! (think relativity, quantum physics, etc.)

Think about that!

(ps. me personally I just cannot stomach that universe is only <15B "years" old. Not saying there was no BANG! (ie. large cosmic event) back then and we're products of it :), but don't think it was "the beginning of everything". Seems very much like "god created universe!" type of thinking)

Vecr
2 replies
4h49m

Why? Sure, maybe a better number to give is "somewhere around 20", but it's difficult to tell how the "true number" could be over 100 or even 50.

miramba
1 replies
4h37m

Explanation needed. How can you, a primitive ape on a tiny remote planet with an average lifespan of 80 years (not meant as an insult, I am too), be sure about what happened 15, 20 or 100 BILLION years ago? I think this is what parent meant with: "people of every time thought they had (almost) complete understanding of the universe." - "Don't think it'll be different this time."

Vecr
0 replies
4h11m

There's good evidence for it, inferred temperature of the early universe included. Obviously not perfect evidence, but even if you assume "tired light", it's another thing all together to come up with a new version of thermodynamics.

physicles
0 replies
28m

Sure, the frontier is always messy. Scientists talk about “quarks” and “dark energy” but those are just placeholders for “we kinda get what they do but we have no idea what they actually are.”

100 or 500 years from now, QM and GR will turn out to be approximations, or shadows, of some deeper theory. But that won’t change how insanely accurate their predictions are.

Our understanding of reality isn’t just moving forward, it’s getting asymptotically closer to ground truth. New theories may upend the conceptual framework, but they still just add decimal points — they have to, otherwise they’re worse theories than what we already have.

pfannkuchen
3 replies
9h5m

I don’t really understand how we can make conclusions about the entire universe when literally all of the data is collecting from one point in space. Couldn’t there be local effects that obfuscate global behavior?

Is there an implicit caveat asterisk on all such statements that is like “as far as we can possibly tell from the data we have” and the reality is we really, really don’t know for sure?

mr_mitm
0 replies
6h10m

“as far as we can possibly tell from the data we have”

Isn't that self evident in all cases?

In cosmology, the implicit assumptions are the Corpernican principle and that GR is correct. That is covered in introductory texts, but unless stated otherwise, it is usually not mentioned, especially not in press releases.

I mean, maybe you're just a Boltzmann brain floating in space and nothing is real. Fun to think about, sure, but expecting to state the basics in any and all cases is unrealistic.

luc4sdreyer
0 replies
8h43m

from one point in space

- We have multiple observatories on and around the planet - The Earth is moving around the Sun - The Sun is moving around the centre of the galaxy - The galaxy is moving towards the great attractor, etc

The Copernican principle states that humans, on the Earth or in the Solar System, are not privileged observers of the universe, that observations from the Earth are representative of observations from the average position in the universe. This has been tested in various ways: https://en.wikipedia.org/wiki/Copernican_principle#Tests_of_...

A nice PBS spacetime video about the topic: https://www.youtube.com/watch?v=q-6oU3jXAho

You can also ask how do we know that the laws of physics haven't changed over time. We don't. But at some point you have to make a few basic assumptions in order to have any hope of making scientific progress.

SkyBelow
0 replies
1h4m

Yes, there is such an implicit caveat on all of science.

Science at its core runs on a repeated process of collect some data, explain it, collect some more data, explain it as well, and keep repeating forever. The scientific model you are likely more familiar with refines this a bit where you use the current model and data to direct where you spend your time/funding collecting more data, but that is mostly a heuristic. Sometimes a person going off on their own will find something amazing to bring back, but we have to remember that many others spend their entire lives and end up finding nothing new or noteworthy.

One problem with science literacy is that too many people treat the existing models as true, even when they aren't. In some cases we know they aren't correct because there are discrepancies in experiments that shouldn't exist, but no model better fits the data and resolves the discrepancies.

At no point can science say "this model is how reality must work". But that is somewhat scary. It is scary to think the truth is unknowable and at best we will have an ever better approximation, so people find it simpler to treat that approximation as truth.

isolli
3 replies
8h11m

[Naive question warning]

What if the cosmological constant (from which, if I understand correctly, the Hubble constant is derived) is not constant? Could a changing comological "constant" explain the discrepancy between the various methods of calculating the Hubble constant?

From Wikipedia [0]: The cosmological constant "was revived and reinterpreted as the energy density of space, or vacuum energy, that arises in quantum mechanics. It is closely associated with the concept of dark energy."

Do vacuum energy and dark energy have to be constant?

[0] https://en.wikipedia.org/wiki/Cosmological_constant

mr_mitm
0 replies
7h10m

Dark energy is a constant in the standard model (equivalent to the cosmological constant) and all observations I'm aware of besides the Hubble tension are consistent with it being constant. But nothing stops you from going beyond the standard model, and people have done that and proposed models with a dynamic dark energy density, for example quintessence models. They'd solve some theoretical issues with the standard model, but so far no observations have been precise enough to differentiate between those models and the standard model.

Having a dynamic dark energy is a proposed solution to the Hubble tension: https://arxiv.org/abs/2103.01183

That is, besides a whole family of potential other solutions:

Dark energy in extended parameter spaces [289] Early Dark Energy [235] Early Dark Energy [229] Dynamical Dark Energy [309] Phantom Dark Energy [11] Decaying Warm DM [474] Metastable Dark Energy [314] Dynamical Dark Energy [11, 281, 309] Neutrino-DM Interaction [506] PEDE [392, 394] GEDE [397] Interacting dark radiation [517] Elaborated Vacuum Metamorphosis [400–402] Vacuum Metamorphosis [402] Self-Interacting Neutrinos [700, 701] IDE [314, 636, 637, 639, 652, 657, 661–663] IDE [314, 653, 656, 661, 663, 670] IDE [656] Self-interacting sterile neutrinos [711] Critically Emergent Dark Energy [997] Unified Cosmologies [747] Generalized Chaplygin gas model [744] f (T ) gravity [814] Scalar-tensor gravity [856] Galileon gravity [876, 882] ¨Uber-gravity [59] Modified recombination [986] Power Law Inflation [966] Reconstructed PPS [978] Super ΛCDM [1007] f (T ) [818] Coupled Dark Energy [650]

kmm
0 replies
7h24m

The Hubble constant is not necessarily derived from the cosmological constant. To be clear, it's not even a constant either, it's a proportionality factor between the recessional velocity of distant objects and their distance from us. Though in some usages it refers to the value of this factor at the current time, which would in fact make it a constant. Regardless, even without a cosmological constant, after the Big Bang you'd still have an expanding universe, possibly collapsing or endlessly expanding, and I like to think of the Hubble constant/parameter in this case as representing the "momentum" the matter in the universe has left from the Big Bang.

The cosmological constant does in fact have to be constant within the constraints of general relativity. The mathematical machinery of GR only allows two parameters: Newton's constant G, representing the coupling of matter to gravity, and the cosmological constant. Both have to be true constants, numbers with a unit.

However, this is only true of the most basic theory of dark energy, where you directly add a constant to the general relativistic lagrangian. More complicated theories, like, for example, quintessence, involve adding new dynamical fields to the theory. The "effective cosmological constant" associated to such theories, quantifying the effect these fields are having on the expansion of the universe, can dynamically change over time, and some of these theories are proposed to solve the Hubble tension. To be clear, although none of these theories are fringe or pseudoscience, they haven't been accepted as a final explanation either, resolving these issues is still a work in progress, at this point they're all simply interesting hypotheses.

harywilke
3 replies
20h3m

Dr.Becky goes over this in a video [0] from a year ago about the divergent results obtained by the two main ways we measure the rate of expansion. Cosmic Microwave Background Vs. Supernovae. As the accuracy of each method has improved, the end results have diverged.

[0] 'theJWST just made the "Crisis in Cosmology" WORSE' https://www.youtube.com/watch?v=hps-HfpL1vc&t=858s

pigpang
2 replies
10h54m

Unfortunatelly, title of article on HN contradicts content of article, so many HN readers skip article because of "confirmed" (in other words: nothing new).

IMHO, a Tired Light theory will better explain facts, but it will require paradigm shift, so there will be a lot of resistance before revolution.

luc4sdreyer
1 replies
8h26m

but it will require paradigm shift, so there will be a lot of resistance before revolution.

I think the main point of resistance is the incompatibility with observations. All Tired light models have been falsified: https://en.wikipedia.org/wiki/Tired_light#Specific_falsified...

If you're aware of a model that can fit some or all of our observations, please share it!

pigpang
0 replies
7h50m

I have my own theory, which is not disproved yet: gravitational background noise slow down light a bit. Gravitation affects whole stream of photons in uniform way, not individual photons. Moreover, it doesn't change direction of photons, so no blur or scattering. My napking maths, which I did few months ago, tells that gravitational delaying should case effect of same magnitude as in red shift, (I did calculation for one frequency only, for proper calculation I need to know the temperature of the noise).

m3kw9
2 replies
19h59m

If we can travel faster than light does it blow up the theory that the universe expands? Because if we can travel faster than light the universe is theoretically be infinite if I can go to a point pas the furthest reaches of stars/matter.

not2b
1 replies
18h19m

If we can travel faster than light, that would blow up all the theories that say that we cannot. And FTL travel would imply time travel (if in your reference frame, something is traveling faster than light, there's another reference frame where it is going backwards in time). A lot of science fiction just sweeps this under the table and pretends that we have Newtonian absolute time to go along with the FTL travel.

m3kw9
0 replies
16h31m

If we can do FTL it may just prove that light speed can be achieved by brute force, or an alternative method. No laws needs to be broken and new laws can be found. Like quantum physics and classical physics.

dbtc
2 replies
17h40m

Can someone help me understand - point me toward some reading or ELI5 - what is the universe expanding into? (Or probably, why is that question not formulated well?)

digging
0 replies
16h57m

The universe is not expanding into anything. It is infinite. However, new space is being created in the voids between objects. It isn't super intuitive or particularly easy to grasp, things are just getting farther apart.

andreareina
0 replies
16h31m

One of the quantities described by Einstein's equations of General Relativity is the metric tensor—a matrix of matrices—that describes how "far" things are in space and time given the stuff in the local environment. One of the things that the equation tells us is that objects that are not gravitationally bound will tend to get farther away from each other as time passes. We call this the expansion of space. As far as we can tell, there isn't anything "outside" the universe is expanding "into"; distances just become larger somehow.

acyou
2 replies
18h58m

Even though the title is copied from the article, we should change the title to "New data indicates the Webb and Hubble telescopes agree on the universe's expansion rate, but not with the cosmic microwave background measurement based expansion rate"

russdill
0 replies
12h8m

Even the article seems confusing at first. The new measurements don't seem to be shedding light on anything or removing any confusion. They just confirm that hubble measurements already made. The Hubble tension remains as confounding a problem as ever.

arbitrage
0 replies
18h42m

That's a mouthful.

The title should be "Hubble Tension almost certainly not caused by measurement error."

ta8645
1 replies
15h33m

Don't know why, but I recently got this gentleman's channel in my feed, who believes that there is no expansion at all:

https://youtu.be/TGpjGVNVYEg?t=397

It's beyond my depth to explain why he's wrong.

pmayrgundter
0 replies
9h58m

Alexander Unziker. Also Eric Lerner, Pierre Robitaille

swamp40
1 replies
2h12m

I cannot for the life of me understand how you look at the cosmic background radiation - which appears equally in every direction you look, for as far away as you can see - and say this is evidence of a "big bang", originating from a single spot at a single point in time. And the universe just "expanded faster than light" to cover everywhere with the same consistent drab layer of cosmic background radiation.

It seems like a child's fairy tale to me.

SkyBelow
0 replies
1h15m

originating from a single spot at a single point in time

That's based on a simplified model of the big bang which isn't what scientist use. It is everywhere, and time itself isn't defined. One major result of this is that many people treat the visible universe as the universe, but those are two different concepts and while the visible universe is finite, the universe might not be (it might be infinite, finite unbounded, or finite unbounded).

Also, science isn't saying "This is what happened". It is saying "Our best model to date says this is happening". In the end it is only a model and the model is still open to being refined and there is always the possibility of data resulting in the entire model being overturned, though often this leads to an ever more complex model that is harder to work with but better fits the experimental data (such as the wave/particle nature of light which is hard for a person to conceptualize).

sudom82
1 replies
16h17m

This article is titled "Webb and Hubble confirm Universe's expansion rate", however I don't see the expansion rate actually listed there, or in the comments. I see a mention of age, but not the rate. Does anyone know the rate? I clicked through to the paper itself[1], but wasn't able to interpret it from the details I could see

[1]: https://iopscience.iop.org/article/10.3847/2041-8213/ad1ddd

russdill
0 replies
12h7m

It seems like a better title would be "Webb confirms Hubble's measurement of Universe's expansion rate"

rmbyrro
1 replies
20h22m

The scale of the universe sometimes feels terrifying to me

swader999
0 replies
20h5m

It's not if you consider we can only move at less than the speed of light or that we go on forever.

merek
1 replies
11h6m

For those interested in the cosmic distance ladder, David Butler has an excellent youtube series "How far away is it?", detailing the methods used to estimate distance along the cosmic distance ladder, the history, and examples. I highly recommend.

https://www.youtube.com/playlist?list=PLpH1IDQEoE8QWWTnWG5cK...

jon_adler
0 replies
10h33m

Thank you. This series of videos is extremely helpful for understanding the concepts referred to in the article.

yen223
0 replies
17h35m

I now really want to know what prompt was used to generate this text

beaned
1 replies
15h28m

Stupid question: how do we know that the universe is strictly exponentially expanding, and not both expanding and contracting in perpetuity like a sin wave? And could such an idea have anything to do with the Hubble tension?

Vecr
0 replies
4h31m

That's the "big crunch" theory. It's been pretty dubious for quite a while now though.

quijoteuniv
0 replies
12h22m

Is it 42?

pantulis
0 replies
9h46m

Play player Euclid.

modelofdemocray
0 replies
16h38m

it's not going to expand faster than what you're able to observe

luxuryballs
0 replies
20h2m

those are some pretty old energy waves

jug
0 replies
6h52m

I can recommend Dr. Becky's introduction to the issues surrounding the Hubble Tension and recent JWST measurements in this 16 minute clip, as she introduces laymen to the problems well: https://www.youtube.com/watch?v=hps-HfpL1vc

It's an older clip, but it checks out... ;) (still JWST discoveries and she most importantly introduces the viewer to the still remaining conflicts)

ChrisMarshallNY
0 replies
20h7m

The thing that strikes me, is the detailed image description. That looks like good fodder for an ML visitor.

3cats-in-a-coat
0 replies
10h41m

The universe is not expanding. The atoms are getting smaller.